#3 Rosh Review

¡Supera tus tareas y exámenes ahora con Quizwiz!

Question: What is the normal ocular pH?

A pH of 7.0 - 7.3 is normal for the eye. Rapid Review Eye Chemical Burn Liquefaction necrosis --> damage to cornea, iris, lens Immediate irrigation (NS/Ringers) until pH is 7.4

Question: What type of precautions are necessary for a patient with active/suspected tuberculosis?

Answer: Airborne precautions.

Question: Which oral antibiotics can cause diarrhea?

Answer: Any oral antibiotic can lead to diarrhea.

Question: How does aspirin inactivate platelets?

Answer: Aspirin inhibits the synthesis of thromboxane A2 (TxA2), a potent stimulator of platelet aggregation.

Question: Which medication class is considered effective for preventing vasospasms following an intracerebral hemorrhage?

Answer: Calcium channel blockers, particularly nimodipine, are effective for preventing vasospasms.

Question: What medical procedure puts uncircumcised boys at risk for developing paraphimosis?

Answer: Catheterization of the urethra during for which the foreskin has to be retracted. Rapid Review Paraphimosis Inability to reduce foreskin over tip of glans secondary to swelling or adhesions Uncircumcised boys at any age, but most commonly at 4 years-old Attempt manual reduction first with pain control (topical) Dorsal slit procedure for failed manual reduction Urological emergency, if untreated, can lead to ischemic injury to the glans

Question: There is a choice between which 2 antibiotics (and what are their durations of therapy) for the treatment of uncomplicated, outpatient acute bacterial prostatitis?

Answer: Ciprofloxacin or trimethoprim/sulfamethoxazole for a minimum of 14 days (often 30 days). Rapid Review Prostatitis Patients <35: N. gonorrhoeae, C. trachomatis Patients >35: E. Coli Fever, chills, perineal/back pain, dysuria Ceftriaxone or ofloxacin and doxycycline x 30 days (patients <35) Ciprofloxacin or TMP/SMX x 30 days (patients >35)

Question: What is the name of the terminal end of the spinal cord?

Answer: Conus medullaris

Question: Which embryologic period is associated with increased teratogenicity during organogenesis?

Answer: Days 21-56 of gestation (3-8 weeks).

Question: What is the effect of cigarette smoking on breastfeeding?

Answer: Decreased milk volume production. Rapid Review Placental Abruption Patient will be a pregnant women in her third trimester With a history of hypertension, trauma, cocaine use Complaining of painful vaginal bleeding Labs will show hypofibrinogenemia

Question: Which value has greater influence on the mean arterial pressure, the diastolic or systolic blood pressure?

Answer: Diastolic blood pressure. Rapid Review Mean Arterial Pressure MAP = (CO x SVR) + CVP MAP = DBP + 1/3 (SBP-DBP) Another way: MAP = [(2 x diastolic)+systolic] / 3

Question: What is Digoxin classified as?

Answer: Digoxin is a positive ionotrope and negative chronotrope. It augments pumping while decreasing heart rate.

Question: What medication can be given as an alternative therapy to benztropine for patients with dystonic reactions?

Answer: Diphenhydramine Rapid Review Dystonic Reaction Antipsychotics, antidopaminergic drugs Hours - days Muscle spasms (ie torticollis) Stiffness Rx: diphenhydramine, benztropine

Question: How do you definitively diagnose myocarditis?

Answer: Endomyocardial biopsy.

Question: Which surgical therapy options are appropriate for achalasia?

Answer: Endoscopic balloon dilation of the lower esophageal sphincter or myotomy with fundoplication.

Question: How is Brudzinski's sign performed?

Answer: Forceful flexion of the neck results in involuntary flexion of the hips, knees, and ankles. Rapid Review Cryptococcal Meningitis HIV, CD4 < 100 Meningismus uncommon Rx: amphoteracin B + flucytosine

Question: Should you order serologic testing to diagnose toxoplasmosis?

Answer: Generally, no, because antibodies are fairly prevalent in the general population. Rapid Review Toxoplasmosis HIV patient Focal neurologic deficits Multiple ring-enhancing lesions Rx: pyrimethamine, sulfadiazine, folinic acid

Question: What genitourinary infection may cause both septic and reactive arthritis?

Answer: Gonorrhea.

Question: What is the most common cause of hyperthyroidism in the United States?

Answer: Graves' disease. Rapid Review Hyperthyroidism Patient will be complaining of heat intolerance, palpitations, weight loss, tachycardia, and anxiety PE will show hyperreflexia Labs will show low TSH and high free T4 Most commonly caused by Graves disease (autoimmune against TSH receptor) Treatment is methimazole or PTU Comments: Propylthiouracil (PTU) P for pregnant

Question: Which types of viral hepatitis lead to hepatocellular carcinoma?

Answer: Hepatitis B and C. Rapid Review Hepatocellular Carcinoma Most common primary liver cancer Most common cause: chronic HBV/HCV cirrhosis Rapidly ↑ ascites Bloody ascitic fluid ↑ AFP

Question: What is the drug of choice for empiric outpatient treatment of bacterial community-acquired pneumonia in children younger than age 5?

Answer: High-dose amoxicillin. Rapid Review Atypical Pneumonia Patient will be complaining of gradual onset of dry cough, dyspnea, and extra-pulmonary symptoms such as headache, myalgias, fatigue, and GI disturbance PE will show rales with auscultation of lung fields Most commonly caused by Mycoplasma pneumoniae Treatment is azithromycin

Question: Which imaging modality is preferred to localize a parathyroid adenoma?

Answer: High-resolution ultrasound.

Question: What is known as a collection of fluid between the parietal and visceral layers of the tunica vaginalis?

Answer: Hydrocele. Rapid Review Testicular Torsion: Patient will be a young male Complaining of intense scrotal pain PE will show exquisite tenderness of the testicle and no cremasteric reflex Diagnosis is made by ultrasound with Doppler Treatment is manual detorsion or surgical

Question: What electrolyte abnormality can be seen in advanced cases of RMSF?

Answer: Hyponatremia. Rapid Review Rocky Mountain spotted fever (RMSF) Patient with a history of recently in the woods hiking or camping Complaining of abrupt onset of severe headache, photophobia, vomiting, diarrhea, and myalgia PE will show maculopapular eruption on the palms and soles Diagnosis is made by skin biopsy Most commonly caused by Rickettsia rickettsia Treatment is ALWAYS doxycycline, even in children

Question: What oral agent can be used to treat scabies?

Answer: Ivermectin. Rapid Review Scabies Patient will be complaining of severe pruritus that is worse at night PE will show small papules, vesicles, and burrows in the webbed spaces of the fingers and toes Diagnosis is made by microscopic visualization Most commonly caused by Sarcoptes scabiei hominis Treatment is permethrin 5%

Question: What lead is most common used for a rhythm strip?

Answer: Lead II. Rapid Review Normal sinus rhythm Rate 60-100 Regular rhythm with P for every QRS PR interval 120-200 QRS 60-100

Question: What liquids have been shown to be effective in drowning otic live insect foreign bodies?

Answer: Lidocaine (spray, gel, and solution), oil (olive and mineral), and alcohol.

Question: Mitral valve prolapse is often associated with what conditions?

Answer: Marfan's syndrome and Ehlers-Danlos syndrome. Rapid Review Mitral Valve Prolapse Thin females Marfan's, Ehlers-Danlos Mid-systolic click with late systolic crescendo murmur ↑ Preload → ↓ murmur; ↓ preload → ↑ murmur Usually asymptomatic

Question: What types of food or drink can bring on an attack of acute gout?

Answer: Meat, seafood, alcohol and foods high in purine.

Question: What are the recommendations for use of metronidazole in pregnancy?

Answer: Metronidazole is an FDA category B medication. Its use during pregnancy is somewhat controverisal. Given the lack of well-controlled studies regarding its safety during pregnancy, it is generally recommended to avoid using the medication in the first trimester.

Question: Does treatment of Strep throat prevent post-streptococcal glomerulonephritis?

Answer: No.

Question: Which wrist ganglion is typically not aspirated?

Answer: One arising on the volar radial wrist, as its proximity to the radial artery makes aspiration risky. Rapid Review Ganglion Cyst Patient will be complaining of a painful mass in wrist Most commonly caused by repetitive activity causing tear or degeneration in joint capsule or tendon synovial sheath Treatment is observation or needle aspiration Comments: most common soft tissue tumors of the hand

Question: Which medications commonly cause urinary retention?

Answer: Opioid analgesics, anticholinergics, antidepressants, antipsychotics and calcium channel blockers. Rapid Review Urge Incontinence Detrussor overactivity Abrupt urge to urinate Rx: anticholinergics

Question: Which pediatric pneumonia is classically associated with a shaggy right heart border?

Answer: Pertussis pneumonia.

Question: What electrolytes are depleted in re-feeding syndrome?

Answer: Phosphate and magnesium. Rapid Review Anorexia Nervosa Patient will be an adolescent female With a history of excessive dieting, purging, body image disturbance, and fear of weight gain Complaining of amenorrhea PE will show > 15% below ideal body weight, lanugo, osteoporosis Treatment is behavioral therapy Comments: Patient will be in denial of illness

Question: What nerve distributions are affected by a Nevus of Ito?

Answer: Posterior supraclavicular and lateral brachiocutaneous nerves.

Question: What is the unique skin feature of Ehlers Danlos Syndrome?

Answer: Skin laxity or loose skin.

Question: What is the most common etiology of bacterial tracheitis?

Answer: Staphylococcus aureus. Rapid Review Laryngotracheitis (Croup) Patient will be a non-toxic appearing child, 6 months to 3 years old Complaining of URI symptoms with barky, seal-like cough, inspiratory stridor, low-grade fever Labs will show steeple sign on PA view Most commonly caused by Parainfluenza virus Treatment is steroids, aerosolized epinephrine

Question: What is the primary method of photoprotection?

Answer: Sun avoidance

Question: What is Russell's sign?

Answer: The presence of calluses on the knuckles that result from induced vomiting in bulimia. Rapid Review Bulimia Recurrent binge eating Over concern with body shape/weight Weight: normal or above normal Submandibular gland enlargement Calluses over dorsal aspect of fingers Dental enamel erosion Rx: cognitive behavioral therapy (first-line), SSRIs (second-line)

Question: Other than peripartum hemorrhage, name a fetal complication of placenta previa.

Answer: The rate of congenital malformations (namely neurologic, cardiovascular, gastrointestinal and respiratory) doubles with placenta previa. Rapid Review Placenta Previa Patient will be a pregnant women in her third trimester Complaining of painless vaginal bleeding Diagnosis is made by transabdominal ultrasound Comments: Do not do a digital vaginal exam

Question: What class of diabetes medications are contraindicated in the setting of heart failure?

Answer: Thiazolidinediones, such as pioglitazone and rosiglitazone. Metformin should also be avoided in patients with decompensated heart failure due to the risk of lactic acidosis. Rapid Review Chronic Heart Failure Treatment Lifestyle modifications Diuretics: used for acute pulmonary edema, no mortality benefit ACEIs: ↓ mortality in all classes BBs: ↓ mortality in classes II, III, IV Hydralazine with nitrates: ↓ mortality in African-Americans Digoxin: used in refractory systolic dysfunction, no mortality benefit Spironolactone: ↓ mortality in class III/IV Isolated diastolic dysfunction: HR and BP control Advanced treatments: mechanical assist device, ICD, heart transplantation

Question: Other than celiac disease, what is the most common autoimmune disorder associated with dermatitis herpetiformis?

Answer: Thyroid disease. Rapid Review Celiac Disease Patient will be complaining of diarrhea, steatorrhea, flatulence, weight loss, weakness and abdominal distension Labs will show IgA anti-endomysial (AGA) and anti-tissue transglutaminase (anti-tTG) antibodies Diagnosis is made by small bowel biopsy Treatment is gluten free diet Comments: associated with dermatitis herpetiformis (chronic, very itchy skin rash made up of bumps and blisters)

Question: Which fungal infection may present with areas of pink or white macules, commonly on the upper torso, that may be confused with vitiligo in patients of dark complexions?

Answer: Tinea versicolor may present as pale macules that do not tan, commonly on the upper trunk. Rapid Review Vitiligo Patient will be complaining of patches of pale skin affecting the neck, upper back and the chest PE will show white, non-scaling, well-demarcated areas of hypopigmentation Most commonly caused by autoimmune destruction of melanocytes Comments: Wood's lamp examination will accentuate the hypopigmentation

Question: Name another common risk factor for gallstones, especially in a patient post abdominal surgery.

Answer: Total parenteral nutrition (TPN). Rapid Review Cholelithiasis Patient will be an obese woman 40 - 50 years old Complaining of slowly resolving right upper quadrant pain that begins suddenly after eating a fatty or large meal Diagnosis is made by ultrasound Most commonly made of cholesterol Treatment is observation or cholecystectomy Comments: Four "F's": Female, Forty, Fat, Fertile

Question: At what ages is the DTaP vaccine recommended?

Answer: Two months, four months, six months, 15 to 18 months and four to six years.

Question: What is the imaging modality of choice for breast abnormalities in the pediatric population?

Answer: Ultrasonography.

Question: What is pseudoesotropia?

Answer: Wide nasal bridge, prominent epicanthal folds or a small interpupillary distance may appear to exhibit esotropia despite normal cover-uncover testing and no evidence of ocular misalignment.

Question: What are the two most common conduction abnormalities seen on ECG with TCA overdose?

Answer: Widened QRS and prolonged QTc.

Question: Can testicular torsion occur during sleep?

Answer: Yes. Up to 50% of all cases are reported to occur during sleep. Rapid Review Testicular Torsion Bimodal: < 1-year-old, puberty Increased risk: undescended testicle, Bell-Clapper deformity Sudden onset of unilateral pain during sleep or exercise Left > right Absent cremasteric reflex Doppler ultrasound Immediate urologic consultation Manual detorsion (medial to lateral/opening a book) if delays expected

A 13-year-old boy is brought to the emergency room by his mother after a suicide attempt. She reports that he swallowed a bottle of pills but is unsure of what he ingested. His vital signs are BP 90/60, HR 135, RR 16, and T 100.3°F. On exam, the patient is somnolent, his pupils are dilated, mucous membranes are dry, and skin is warm and flushed. Which of the following substance did this patient most likely ingest? Amitriptyline Methylphenidate Oxycodone Sertraline

Correct Answer ( A ) Explanation: Amitriptyline is a tricyclic antidepressant (TCA) and the most frequent cause of poisoning in this class of drugs. Overdose can lead to a variety of anti-cholinergic symptoms, including tachycardia, dry mouth, mydriasis, hyperreflexia, warm flushed dry skin, gastrointestinal complaints, urinary retention, and confusion or agitation. Severe overdose can cause life-threatening symptoms such as coma, seizure, dysrhythmia, and cardiac arrest. Initial management may include gastric decontamination with activated charcoal but is otherwise supportive, depending on the presenting symptoms (intravenous fluids, cardiac agents, respiratory support). Methylphenidate (B) is an amphetamine, and overdose leads to sympathomimetic symptoms such as tachycardia, hypertension, hyperthermia, psychosis, seizures, and mydriasis. Oxycodone (C) is an opiate and can lead to coma and respiratory depression if taken in high doses. Pinpoint pupils and hyporeflexia can also be seen. Sertraline (D) is a selective serotonin reuptake inhibitor and has a wide therapeutic window, so overdose typically does not cause life-threatening symptoms. However, vomiting, mild CNS depression, or tremor can occur.

Which of the following conditions is associated with celiac disease? Dermatitis herpetiformis Erythema marginatum Pemphigus vulgaris Stevens-Johnson syndrome

Correct Answer ( A ) Explanation: Dermatitis herpetiformis is a dermatologic condition associated with celiac disease. It is an uncommon autoimmune, cutaneous eruption that occurs as a manifestation of gluten sensitivity. Affected patients typically develop intensely pruritic inflammatory papules and vesicles on the forearms, knees, scalp, or buttocks. Genetic predisposition and gluten-sensitivity are key factors in the pathogenesis and virtually all patients carry the HLA DQ2 or HLA DQ8 haplotype. Antibodies against epidermal transglutaminase produced in association with an immune response to ingested gluten play a key role in the disease pathogenesis. Dapsone and a gluten-free diet are the primary interventions for the management of this disease. Dermatitis herpetiformis is usually a life-long condition that requires continued treatment. Erythema marginatum (B) is an evanescent, pink or faintly red, non-pruritic rash involving the trunk and sometimes the limbs that is associated with acute rheumatic fever. Pemphigus vulgaris (C) is an autoimmune disorder characterized by autoantibodies to adhesion molecules in the epidermis which cause bullae to develop, which rupture and lead to widespread erosions and desquamation. Stevens-Johnson syndrome (D) is a severe mucocutaneous reaction characterized by extensive necrosis and detachment of the epidermis. This is a rare but serious side effect of certain medications and requires hospitalization.

A patient presents after a chemical splash to the eye. What management is immediately indicated? High-volume irrigation Pupil dilation and slit lamp examination Referral to ophthalmology Topical antibiotics and ophthalmology consultation

Correct Answer ( A ) Explanation: Patients who present with chemical exposure to the eye should undergo immediate irrigation with normal saline prior to the initiation of any other management. A rapid assessment of the pH of the conjunctiva with pH paper and application of a topical anesthetic may be rapidly performed prior to initiation of irrigation but no procedure or intervention should delay irrigation. Irrigation should continue for a minimum of 30 minutes followed by a check of the pH. Irrigation should be continued until the pH normalizes. Once the pH normalizes, application of a cycloplegic, topical antibiotics and a complete ocular assessment should be performed. After immediate management, focus should be placed on identifying the substance the patient was exposed to and ophthalmologic consultation for any significant injuries. Typically, alkaline substances with a pH less than 12 and acidic substances with a pH greater than 2 are not thought to cause significant injury. This may be altered if the duration of contact was prolonged. Long-term complications include perforation, scarring and neovascularization of the cornea. Glaucoma and cataracts can also occur. Although an ophthalmology assessment (C) is necessary, it should not delay irrigation and normalization of pH as this is the most vital intervention to prevent further damage. Topical antibiotics (D) will likely be needed if there is a corneal abrasion or ulceration but should not be applied until the pH is normalized. Pupil dilation and slit lamp examination (B) is time consuming and should be deferred until irrigation is completed.

A 59-year-old man presents with 2 days of dysuria, frequency, lower abdominal pain, and chills. He has been eating and drinking normally and is able to urinate, though he has general malaise. He denies penile discharge, testicular pain, sexual activity, a history of sexually transmitted infections, prior urinary symptoms, or weight loss. The patient's abdomen is soft and nontender; his genitourinary exam is normal except that on a gentle rectal exam, he has a boggy, tender prostate. A urinalysis, urine culture, and Gram stain are sent and pending. Which of the following is the most likely cause of this patient's symptoms? Escherichia coli Neisseria gonorrhoeae Peptostreptococcus magnus Pseudomonas aeruginosa

Correct Answer ( A ) Explanation: This is a patient with acute bacterial prostatitis, as evidenced by the development of acute urinary symptoms, malaise, chills, and a boggy, tender prostate. Most of the causative organisms are the same as those that cause urinary tract infections, and E. coli is responsible for more than 80% of cases. Because this patient is over the age of 35 and has no history of sexually transmitted infections, recent contacts, or penile discharge, he is less likely to have N. gonorrhoeae (B), which is much more common in younger men. Gram-positive bacteria, of which P. magnus (C) is an example, are extremely rare causes of prostatitis with the exception of Enterococcus spp. Although P. aeruginosa (D) is often found in prostatitis, it is much less common than E. coli.

A 44-year-old woman presents with involuntary movements of the neck, face and tongue. She reports that she was seen earlier in the Emergency Department for vomiting and received metoclopramide. What management is indicated? Benztropine intramuscular Lorazepam intramuscular MRI of the brain Noncontrast CT scan of the head

Correct Answer ( A ) Explanation: This patient presents with a dystonic reaction after the administration of metoclopramide and should be given benztropine. Acute dystonia is the most common adverse effect seen after the administration of neuroleptic agents and can occur in up to 5% of patients. It can also occur with medications that antagonize central and peripheral dopamine receptors such as metoclopramide. This reaction is caused by disruption in the balance between dopaminergic and cholinergic pathways in the basal ganglia. They tend to occur within 48 hours of administration. Patients often experience tongue protrusion, acute torticollis, sustained eye deviation (usually upwards) and arching of the back. Laryngospasm is rare but potentially life threatening. Because dystonic reactions occur as a result of cholinergic dominance, they should be treated with anticholinergic agents including benztropine and diphenhydramine. These agents typically reverse symptoms immediately. Lorazepam (B) may reduce symptoms but will not rapidly reverse them. Neuroimaging (C & D) is not necessary for the diagnosis or treatment of a dystonic reaction.

A 15-year-old girl presents to urgent care with a nonproductive cough and low-grade fever that developed insidiously over the previous week. Physical exam reveals diffuse rales on pulmonary auscultation, and chest X-ray shows diffuse non-focal infiltrates. Which of the following is the most accurate statement regarding this patient's condition? Extrapulmonary manifestations may include hemolysis and CNS involvement Mycoplasma pneumoniae is the most common etiology in all age groups Penicillin is the most appropriate treatment The formation of cold agglutinins is a positive confirmatory test

Correct Answer ( A ) Explanation: This patient's clinical presentation is characteristic of community-acquired pneumonia. There are numerous pathogens that cause community-acquired pneumonia in children with a predilection for causing disease in particular age groups. Mycoplasma pneumonia, Chlamydophila pneumonia, and S. pneumoniae are most common in school age through the teen years. S. pneumoniae is classically associated with "typical" pneumonia, which is marked by rapid onset of high fever accompanied by chills, a high white count (>15,000), and findings of lobar consolidation on chest X-ray. M. pneumoniae and C. pneumoniae are classically associated with "atypical" pneumonia and present with a more gradual onset of symptoms and a pattern of interstitial infiltrates on chest radiography. Atypical bacterial infections are also more likely to present with extrapulmonary manifestations such as skin rashes, hemolysis (secondary to the formation of cold agglutinin antibodies), CNS involvement (aseptic meningitis, cerebellar ataxia, cranial nerve palsies), and gastrointestinal symptoms. However, these typical and atypical features frequently overlap and cannot be used consistently to distinguish between different etiologic agents. While Mycoplasma pneumoniae and Chlamydophila pneumoniae are more common in children older than age five (particularly in middle and high school age students), Streptococcus pneumonia (B) is the most common bacterial cause of pneumonia in children overall. Viruses are more common in those younger than five years, accounting for up to fifty percent of cases in young children. The formation of cold agglutinins (D) is a nonspecific finding of IgM antibodies reacting to the erythrocyte I antigen and is neither sensitive nor specific for detecting the presence of M. pneumoniae in children. The most appropriate empiric treatment when M. pneumoniae or C. pneumoniae is suspected is a macrolide antibiotic such as azithromycin or erythromycin (C). Penicillin is not effective against M. pneumonia.

A 46-year-old man presents with right eye pain and a change in his vision. He was recently diagnosed with tuberculosis and started on a 4-drug regimen. Based on his history and physical exam findings, you are concerned for optic neuritis. Which of the following medications is likely responsible for his symptoms? Ethambutol Isoniazid Pyrazinamide Rifampin

Correct Answer ( A ) Explanation: Tuberculosis (TB) is the world's second leading cause of infectious death, with more than 8 million people acquiring active TB each year and nearly 2 million deaths. There was a reemergence of tuberculosis in the 1980's due in large part to the epidemic of HIV/AIDS. However, rates of tuberculosis in the US have declined steadily since that time. Of serious concern, is the emergence of multidrug resistant strains of Mycobacterium tuberculosis which guides the initial management of patients with TB. One of the basic principles in treatment of tuberculosis is that regimens must include multiple drugs to which the possible resistant organism is susceptible. First-line agents include isoniazid, rifampin, pyrazinamide and ethambutol. The preferred regimen consists of these four medications administered for eight weeks, followed by isoniazid and rifampin for an additional eighteen weeks. Ethambutol can cause optic neuritis resulting in decreased visual acuity and possible blindness. The medication should be discontinued immediately at the first sign of visual loss. Because of the difficulty in monitoring vision in children, ethambutol is not recommended in this population. Side effects of isoniazid (B) include hepatitis and peripheral neuropathy, the latter of which can be prevented with coadministration of pyridoxine. Pyrazinamide (C) may cause hepatotoxicity and polyarthralgias. Rifampin (D) causes an orange discoloration of body fluids, including urine, tears, sweat and sputum.

A 59-year-old woman with a history of poorly controlled hypertension presents to the emergency room for a sudden, very severe headache that began 50 minutes ago. She is disoriented and vomiting. A CT of her head showed no acute abnormalities. Which of the following is the next best in her management? Electroencephalogram Lumbar puncture MRI head Supportive care

Correct Answer ( B ) Explanation: A lumbar puncture is the next best step to rule out a spontaneous subarachnoid hemorrhage (SAH). This patient has exhibited the classic symptoms of a SAH: a sudden headache, often of severity never experienced previously, nausea and vomiting, and impairment of consciousness that may progress to coma. A physical exam can show nuchal rigidity. Focal neurologic signs usually only present in the setting of a focal intracerebral hematoma. The patient's history might reveal one or more risk factors for intracerebral hemorrhages, including recent head trauma, hypertension, tobacco use, or heavy alcohol use. Females of advancing age are at the greatest risk. At the first suspicion of a SAH, a CT scan, with CT angiography if possible, should be performed. If the CT scan does not confirm a hemorrhage, a lumbar puncture must be performed to fully rule out the diagnosis. An intracerebral hemorrhage is suspected when the cerebrospinal fluid is bloodstained in a quantity greater than that of a traumatic tap. Xanthochromia is also indicative of cerebral bleeding. Once diagnosed, patients need admission, and cerebral angiography should be done to determine the source of bleeding. Surgical clipping or endovascular treatment by interventional radiologists is necessary to definitively treat the hemorrhage. Following this, measures to prevent rehemorrhaging during the critical days following an intervention are necessary. These include bed rest with avoidance of physical exertion, and use of antiemetics, stool softeners, and antitussives as applicable. Seizure prophylaxis, usually with phenytoin, may also be used. An electroencephalogram (EEG) (A) will not allow for visualization of an acute hemorrhage. An EEG is generally more useful in making the diagnosis of seizures and in determining brain activity in patients with altered mental status, which are not acute concerns in this patient. An MRI of the head (C) is incorrect, as this imaging usually takes a significantly longer time than a CT scan. Additionally, CT scans are often more sensitive for diagnosing intracerebral hemorrhages within the first 24 hours than MRI scans. Supportive care (D) is important to consider if the diagnosis of an intracerebral hemorrhage, or equally critical condition, is definitively ruled out. At that point, clinicians may consider other, less life-threatening causes of head pain, such as migraine headaches, and provide support as applicable.

Which one of the following cardiac rhythm abnormalities is most common in patients with anorexia nervosa? Atrial fibrillation Sinus bradycardia Sinus tachycardia Ventricular fibrillation

Correct Answer ( B ) Explanation: Anorexia nervosa is characterized by restriction of food intake resulting in low body weight, intense fear of gaining weight or becoming fat, and disturbance of body image. It occurs most often in adolescent females and is often accompanied by depression and other comorbid psychiatric disorders. For low-weight patients with anorexia nervosa, virtually all physiologic systems are affected, ranging from hypotension and osteopenia to life-threatening dysrhythmias, often requiring emergent assessment and hospitalization for metabolic stabilization. Sinus bradycardia is almost universally present in patients with anorexia nervosa. It is hypothesized that this is due to vagal hyperactivity resulting from an attempt to decrease the amount of cardiac work by reducing cardiac output. As cachexia progresses, patients with anorexia nervosa lose strength and endurance, move more slowly, and demonstrate decreased performance in sports. Overuse injuries and stress fractures can occur. Bradycardia, orthostatic hypotension, and palpitations may progress to potentially fatal dysrhythmias. The focus of initial treatment for patients who have anorexia nervosa with cachexia is restoring nutritional health, with weight gain as a surrogate marker. Feeding tubes may be needed in severe cases when the patient has a high resistance to eating. Refeeding syndrome can occur in a malnourished individual when a rapid increase in food intake results in dramatic fluid and electrolyte shifts, and is potentially fatal. Sinus tachycardia (C) may occur with refeeding in patients with anorexia. Atrial fibrillation (A) is the most common cardiac arrhythmia. Atrial fibrillatrion can be attributed to underlying conditions such as alcoholism, hyperthyroidism, substance abuse and other. Ventricular fibrillation (D) is the most commonly identified arrhythmia in cardiac arrest patients. Coronary artery disease is the single most common etiologic factor.

A 16-year-old girl is seen in clinic due to a concern for a breast mass that she notes has been present for 3 months. The size of the mass has not changed over the previous 3-months. She is otherwise healthy and is not on any medications. On exam you note a 2 cm diameter mass on the right outer quadrant of her right breast that is rubbery, mobile, and not tender. Which of the following is the most likely diagnosis? Cystosarcoma phylloides Fibroadenoma Fibrocystic change Metastasis

Correct Answer ( B ) Explanation: Breast masses in adolescent girls are typically benign, represented mostly by fibroadenomas and fibrocystic changes. Breast malignancies are exceedinlgly rare in this age group. The patient most likely has a fibroadenoma which is the most common breast mass in adolescents and young adults. The peak incidence is between 25 and 40 years and decreases after age 40. Fibroadenomas are characterized by a painless, firm, solitary, mobile, breast mass. Fibroadenomas are partially hormone-dependent and may enlarge in pregnancy. They often involute at menopause. The lesions are not fixed to the surrounding parenchyma and slip under the palpating hand; this has led to fibroadenomas being referred to as a breast "mouse." Because there is a very small risk for malignancy, these patients should undergo monitoring for changes in size of the mass, have an ultrasound, or fine needle aspiration. Treatment is supportive, but sometimes requires surgical excision. Cystosarcoma phylloides (A) can occur in adolescents and can mimic a fibroadenoma. However, the tumor often grows rapidly and can become relatively large. Fibrocystic change (C) usually varies in size during the course of a menstrual cycle. It can present with pain and often is found bilaterally. Metastasis (D) can occur from a primary malignancy. However, this patient has no constitutional symptoms or suspicion for metastatic disease.

Which of the following is consistent with a diagnosis of bulimia? Congestive heart failure Contraction alkalosis Hyperkalemia Hypothermia

Correct Answer ( B ) Explanation: Bulimia is associated with a number of signs and symptoms as well as laboratory abnormalities. Bulimia is often a more difficult diagnosis to make than anorexia as patients with anorexia have significant weight change while those with bulimia often maintain a normal weight. Patients with eating disorders have high rates of other psychiatric comorbidities. Major depression is seen in up to 80% of patients with anorexia. Substance abuse is more common in bulimia (30-70%) than in anorexia (12-18%). Patients with eating disorders present with vague symptoms including fatigue, dizziness, nausea and generalized weakness. Vomiting in bulimia can cause dysphagia, sore throat and hematemesis. Gastric acid can cause tooth decay from enamel erosion. Additionally, vomiting can cause contraction alkalosis. Hypokalemia, not hyperkalemia (C) is associated with bulimia. Congestive heart failure (A) and hypothermia (D) are seen in anorexia due to caloric restriction.

How do you calculate mean arterial pressure (MAP)? [DBP = diastolic blood pressure, SBP = systolic blood pressure] MAP = [DBP + (2 x SBP)]/3 MAP = DBP + 1/3(SBP-DBP) MAP = DBP + 2/3(SBP-DBP) MAP = SBP + 1/3(SBP-DBP)

Correct Answer ( B ) Explanation: Calculation of the mean arterial pressure (MAP) provides a weighted average of the systolic blood pressure (SBP) and the diastolic blood pressure (DBP). It is a determination of tissue perfusion and is normally 70-100 mm Hg in adults. A MAP of approximately 60 is necessary to perfuse the coronary arteries, brain, and kidneys. The MAP is calculated by either formula: MAP = [DBP + (2 x SBP)]/3 (A); MAP = DBP + 2/3(SBP-DBP) (C); and MAP = SBP + 1/3(SBP-DBP) (D) are incorrect formulas.

A tax accountant presents with wrist pain. For the past year, she has had an intermittent mass that appears on the dorsum of her wrist. It comes and goes, but never seems to be the same size. Recently, with the increased workload of tax season, it has persisted for the past 3 weeks. Examination reveals a nonpulsatile, mildly tender mass on the lateral wrist dorsum. There is no surrounding erythema or edema. It transilluminates when a penlight is placed next to it. Percussion of this mass does not produce distal paresthesias. Which of the following is the most likely diagnosis? Aneurysm Ganglion Lipoma Neuroma

Correct Answer ( B ) Explanation: Ganglia are the most common soft tissue tumors of the hand. They are cystic structures which arise from a torn or degenerated joint capsule or tendon synovial sheath. Basically, they are one-way-valved "outpocketings" of synovium, connected to the capsule proper and filled with synovial fluid. The most common locations are the wrist dorsum, the volar radial wrist and the base of the finger at the proximal annular ligament of the flexor tendon sheath. They may arise and resolve spontaneously, but are most common during an increase in repetitive activity. A painful mass is a common complaint. In some cases, the ganglion is large enough to compress nearby nerves, leading to median or ulnar sensorimotor deficits. Ganglia are usually more painful and more prominent with flexor or grasping activity. Radiographs should be obtained in all cases to rule out bony pathology. Treatment includes immobilization with orthoses, aspiration or surgical excision. Aneurysms (A), especially of the distal radial artery, can be confused with wrist ganglia. However, aneurysms will pulsate, and ganglia typically do not. Solid tumors, like lipomas (C), typically do not transilluminate when a penlight is placed at its side. Neuromas (D) are exquisitely tender, and typically produce radiating sensory symptoms when struck or compressed.

Which of the following is true regarding primary hyperthyroidism? Lithium is a common cause Propranolol is the drug of choice for adrenergic symptoms TSH is elevated Weight gain is common

Correct Answer ( B ) Explanation: Hyperthyroidism is a condition in which there is overproduction and increased circulation of thyroid hormone. Hyperthyroidism has a variety of causes and variable presentation. Increased circulating thyroid hormone causes a hypermetabolic state and increases beta-adrenergic activity. Initially, patients may have vague constitutional symptoms. As the disease progresses, clinical manifestations may become more organ-specific. Thyrotoxicosis or thyroid storm represents the most severe manifestation of the disease. Thyroid storm is life threatening and characterized by hyperadrenergic activity. Patients present with vital sign abnormalities including tachypnea, tachycardia, hyperthermia and hypertension. ECG may show atrial dysrhythmias like atrial flutter and fibrillation or simple sinus tachycardia. High-output cardiac failure is common as well. Physical features include goiter, opthalmopathy and tremors. Patients will also have increased reflexes and altered mental status. Thyroid storm treatment involves suppression of thyroid hormone synthesis and secretion, prevention of peripheral conversion from T4 to T3 and blocking the peripheral adrenergic stimulation. Blocking the peripheral effects of thyroid hormone is best accomplished with a beta-blocker and propranolol is preferred as it also decreases conversion of T4 to T3. Lithium (A) is a cause of hypothyroidism. In primary hyperthyroidism, TSH is depressed (C). TSH may be elevated in cases of TSH-secreting pituitary adenomas, although this is a rare cause of hyperthyroidism. Weight gain (D) is common in hypothyroidism.

An inotropic drug works by which of the following mechanisms? Alter cardiac conduction velocity Alter the force of myocardial contraction Alter the heart rate Dilate blood vessels

Correct Answer ( B ) Explanation: Inotropic drugs alter the force of myocardial contraction. Inotropic, chronotropic and dromotropic are different classifications of cardiac drugs. Chronotropic drugs effect the heart rate and dromotropic drugs affect the conduction velocity though the myocardium. These cardiovascular drugs differ from each other primarily on the basis of the aspects related to cardiac performance which they influence or affect. There are two types of inotropic agents- positive and negative. Positive inotropic agents increase the force of myocardial contraction by increasing intracellular calcium concentrations. Increased contraction results in increased stroke volume and cardiac output for each ventricular filling. These agents are used to support impaired systolic function in conditions such as decompensated systolic heart failure, cardiogenic shock, septic shock and myocardial infarction. Examples include dopamine, dobutamine, norepinephrine, epinephrine and milrinone. Negative inotropic agents decrease the force of myocardial contraction and are used to decrease cardiac workload. This can be useful in different clinical situations such as angina or hypertrophic cardiomyopathy. Negative inotropic agents include beta blockers, calcium channel blockers, quinidine, and flecainide. Chronotropic drugs alter the heart rate (C). They are further classified into positive and negative chronotropic agents. Positive chronotropic drugs increase the heart rate by acceleration the rate of impulse formation in the SA node. Negative chronotropic drugs slow down the heart rate by decreasing impulse formation. Dromotropic drugs alter the conduction velocity (A). Positive dromotropic drugs increase the rate of conduction, whereas negative dromotropic drugs decreases the rate of electrical conduction. Vasodilators dilate blood vessels (D) and include drugs such as nitrates, calcium channel blockers and angiotensin converting enzyme inhibitors

A woman presents with right breast pain, fever, and malaise for 3 days. She has been breastfeeding her newborn child for the last 3 weeks. On exam, there is an area of focal erythema and tenderness. No mass or fluctuance is noted. What is the most likely pathogen responsible for causing her condition? Escherichia coli Staphylococcus aureus Streptococcus agalactiae Streptococcus pyogenes

Correct Answer ( B ) Explanation: Mastitis is a localized and painful inflammation of the mammary gland that can be associated with fever and malaise. Lactational mastitis primarily occurs within the 1st few months of breastfeeding when the skin of the breast is prone to damage due to frequent feedings. It may also occur much later when the infant develops teeth and can cause local trauma to the area during feeding. The most common pathogen is Staphylococcus aureus, which accounts for 40% of the cases. It is important to distinguish mastitis (cellulitis) from an abscess (requires surgical drainage) and inflammatory breast cancer (rare, but deadly). In addition to antibiotics such as dicloxacillin or cephalexin that cover for Staphylococcus aureus, the patient should be encouraged to apply warm compresses and continue breastfeeding. If the patient does not respond to antibiotics within 72 hours, the patient should be evaluated again for the possibility of breast abscess. Rapid review, Mastitis: Lactating women 2-3 weeks postpartum Staphylococcus > Streptococcus, H. Influenza Instruct mothers to keep nursing on side of infection First generation cephalosporin or beta-lactamase resistant PCN Escherichia coli (A) and Streptococcus species, such as Streptococcus agalactiae (C) and Streptococcus pyogenes (D) have been noted as causes of mastitis as well, but less frequently than Staphylococcus aureus.

A 19-year-old woman presents with four days of high fever, headache, myalgias. She also developed blanching erythematous rash with macules on her wrists and ankles that spread to her trunk. She denies history of distant travel but was camping last weekend at a local state park in North Carolina. Which of the following is the most appropriate treatment? Ceftriaxone Doxycycline Erythromycin Vancomycin

Correct Answer ( B ) Explanation: Rocky Mountain spotted fever (RMSF) is a tick-borne illness caused by Rickettsia rickettsii. Following a tick bite, the incubation period ranges from two days to two weeks. Patients will present with an abrupt onset of fever, severe headache, myalgias, and nausea and vomiting. Fever is nearly always present in the first few days of illness and may precede other symptoms by a week or more. The rash is the result of vascular endothelial damage and usually appears on the third to fifth febrile day of the illness. The rash is seen first on the wrist and ankles (with involvement of the palms and soles in 50% of cases) and initially is a blanching, nonpalpable, discrete macular rash. Within hours, the rash spreads to the extremities and trunk. Within a few days, the rash becomes petechial and no longer fades. Patients will usually have a normal white blood cell count, but may be thrombocytopenic. Treatment includes doxycycline (even for children) and supportive care. Chloramphenicol is an alternative antibiotic that can used in patients with severe allergy or pregnant women. Ceftriaxone (A), erythromycin (C), and vancomycin (D) are not appropriate antibiotics for Rocky Mountain spotted fever.

A three-year-old girl presents with a diffuse maculopapular rash that is most prominent in the bilateral axillae. The rash is pruritic, worse at night, and excoriations are present over the extremities. What is the treatment of choice? Hydrocortisone Permethrin Reassurance Topical emollients

Correct Answer ( B ) Explanation: The child's presentation is most concerning for scabies. Scabies is caused by the mite Sarcoptes scabiei. It's incubation period is three to six weeks after the time of acquisition through direct contact. At this time, the affected individual develops an intensely pruritic, maculopapular rash. Individual lesions may also develop a fine crust. Lesions may also result in burrows, 2 to 15 mm linear streaking that results from movement of the mite within the skin. The rash classically involves the interdigital webs, wrist flexors, elbow and knee extensors, lower buttocks and thighs, and axillae. The treatment of choice is application of topical permethrin from the neck down. The cream should be left on overnight and be removed by washing eight to 14 hours later. All family members should be treated regardless of current symptomatology because re-infestation is otherwise very common. An alternative treatment of scabies is lindane, but due the risk of toxicity, lindane is recommended by the US CDC only for those who cannot tolerate topical permethrin. Linens should be washed in hot water and then dried on high heat. Items that cannot be washed should be placed in plastic bags for at least three days so that the mites will die. Hydrocortisone (A) and topical emollients (D) are used to treat atopic dermatitis but do not treat scabies. Topical steroids should only be given to patients with scabies AFTER mite eradication to control itching, which may persist for one to two weeks after adequate treatment. Oral antihistamines can also be used to control itching. Reassurance (C) is not appropriate, as scabies is unlikely to self-resolve and is extremely contagious.

A 22-year-old woman is undergoing treatment for diarrhea that began after broad-spectrum antibiotic therapy for sepsis. The diarrhea is improving on appropriate antibiotics, and she will be discharged home to complete her oral therapy. Which of the following should she be specifically instructed to avoid? Dairy products Ethanol Excessive water intake Leafy green vegetables

Correct Answer ( B ) Explanation: The patient developed diarrhea during treatment with broad-spectrum antibiotics, which is concerning for alteration of the intestinal microbiome with subsequent overgrowth and infection with Clostridium difficile. The drug of choice to treat C.difficile infections is metronidazole, which is both effective and well-tolerated. However, patients should be specifically instructed to avoid alcohol intake, as even a small amount of alcohol ingestion during metronidazole therapy may result in intense vomiting, also known as a disulfiram-like reaction. Disulfiram is a medication used to treat alcoholism by inhibiting the complete breakdown of ethanol. When acetaldehyde, a byproduct of ethanol metabolism, accumulates in the blood, it causes nausea, vomiting, flushing, palpitations, headache, and lowered blood pressure. This constellation of symptoms constitutes the disulfiram reaction. Dairy products (A) should not be consumed at the same time as tetracyclines. Co-ingestion of dairy and metronidazole is not contraindicated. Excessive water intake (C) does not need to be avoided while on metronidazole. Adequate oral intake should be encouraged to assure hydration in the setting of C.difficile colitis. Leafy green vegetables (D) are not to be avoided during metronidazole therapy. However, leafy green vegetables contain high amounts of vitamin K, and dietary intake needs to be carefully monitored and regulated in patients on warfarin, which is a vitamin K inhibitor.

Which of the following is the most appropriate treatment for urge incontinence? Estrogen replacement therapy Oxybutynin Pseudoephedrine Sildenafil

Correct Answer ( B ) Explanation: Urinary incontinence is characterized by the unintentional loss of urine. Urge incontinence is the most common type of incontinence in geriatric patients and is due to detrusor overactivity. Patients may feel an abrupt urge to urinate, but cannot get to the toilet in time. It is also associated with nocturia. The workup begins with a bladder-voiding diary, urinalysis and urine culture. Examination should begin with determining post void residual (PVR) by having the patient completely void, then checking for residual urine volume via catheterization or ultrasonography. PVR > 200 indicates urinary retention. Nonpharmacologic therapy should be tried first. These include frequently scheduled voids, bladder training such as urge suppression, and behavior modifications such as reducing caffeine and alcohol and late night fluid intake. Pharmacologic therapy includes anticholinergics such as oxybutynin and tolterodine. These agents antagonize acetylcholine at muscarinic receptors, leading to bladder smooth muscle relaxation. Tolterodine may cause less dry mouth than oxybutynin. Estrogen replacement therapy (A) is used in stress incontinence (outlet incompetence). Pseudoephedrine (C) is an alpha-adrenergic agonist that increases urethral tone and used in stress incontinence. Sildenafil (D) is used in the treatment of erectile dysfunction, not urinary incontinence.

A 27-year-old woman presents to her primary care physician with dysphagia. Her symptoms began several months ago. She has trouble swallowing solids and liquids though liquids seem to make her choke and sputter the most; therefore, she has been unable to eat and has thus experienced significant weight loss. She has no significant past medical history apart from a 20-pack-per-year smoking history. She denies any recent travel. Physical examination is within normal limits. A barium esophagram reveals a "bird's beak" appearance. What is the next best step in management? Begin a calcium channel blocker Begin botulinum toxin injections Esophageal manometry Upper endoscopy

Correct Answer ( C ) Explanation: Achalasia is a motor disorder of the distal esophagus resulting from degeneration of Aurbach's plexus. It is the most common motility disorder and is often found in patients younger than 50 years of age. The lower esophageal sphincter fails to relax during swallowing. As a consequence, natural peristalsis is disrupted and the patient experiences dysphagia to solids and liquids, with liquids often being most problematic. A barium esophagram is helpful in making the diagnosis and should reveal the classic bird's beak tapering at the esophageal sphincter. If a barium swallow is suggestive of achalasia, esophageal manometry should be performed to confirm the diagnosis. Esophageal manometry will reveal elevated resting lower esophageal sphincter pressure, incomplete lower esophageal sphincter relaxation after swallowing, and almost total absence of peristalsis in the esophageal body. After the diagnosis of achalasia is made with a barium esophagram and an esophageal manometry, one must perform an upper endoscopy (D) prior to beginning medical or surgical management. Endoscopy is the gold standard for properly ruling out malignancy. Her smoking history and history of weight loss are both red flags which need to be evaluated in a timely fashion. Treatment includes medical management consisting of calcium channel blockers (A), and botulinum toxin (B) injections.

A patient with AIDS presents with fever, severe headache, and positive Kernig's sign. CSF findings reveal lymphocytosis and increased protein levels. India ink staining reveals encapsulated yeast forms. Which of the following is the most likely pathogen? Borrelia burgdorferi Candida albicans Cryptococcus neoformans Neisseria meningitidis

Correct Answer ( C ) Explanation: Although all of the above pathogens can cause meningitis in a patient with AIDS, Cryptococcus neoformans has the following spinal fluid findings, including increased opening pressure, variable pleocytosis, increased protein, and decreased glucose. Gram stain of the cerebrospinal fluid usually reveals budding, encapsulated fungal cells. Prior to waiting on Gram-stain results, the CSF can be sent for detection of the cryptococcal antigen (highest sensitivity in AIDS patients). Detecting a serum cryptococcal antigen also aids in the diagnosis. Treatment of cryptococcal meningitis involves antifungal therapy such as amphotericin B and flucytosine, followed by fluconazole suppression until adequate immune reconstitution, or if this does not occur, indefinitely. Neisseria meningitidis (D) typically presents with fever, confusion, and petechial rash of skin and mucous membranes, and is most commonly found in populations living in close quarters, such as college freshmen. Although Candida albicans (B) is also a fungal source of meningitis, india ink staining is specific to Cryptococcus infections. To diagnose Lyme meningitis from Borrelia burgdorferi (A), serum enzyme-linked immunosorbent assay (ELISA) testing with confirmatory western blot is recommended.

A 68-year-old woman is diagnosed with hyperparathyroidism. Both her serum calcium level and parathyroid hormone level are elevated. Which of the following findings would be most consistent with the diagnosis? Chvostek's sign Decreased serum magnesium level Decreased serum phosphate level Hypotension

Correct Answer ( C ) Explanation: Calcium and phosphate have an inverse relationship with one another. As serum calcium levels increase, an increase in parathyroid hormone (PTH) levels will promote the renal excretion of phosphate, causing serum levels of phosphate to decrease. Hyperparathyroidism is the most common cause of hypercalcemia and can be classified as primary, secondary, or tertiary. Primary hyperparathyroidism is typically due to a defect in the parathyroid glands that cause PTH release in the setting of elevated calcium levels. Normally, additional PTH release is inhibited when serum calcium levels rise. Causes include parathyroid cancer, adenoma, or dysfunctional calcium sensing receptors on the surface of chief cells. Other lab abnormalities include serum hypophosphatemia and an increased urine calcium level. Surgical intervention with a parathyroidectomy is the mainstay of therapy. Chvostek's sign (A) can be present in hypocalcemia. Serum magnesium (B) is directly related to serum calcium levels, not inversely related. Hypertension, not hypotension (D), can be related to hypercalcemia.

A 17-year-old girl is examined for a routine visit. She eats a healthy diet. She also stays active by playing volleyball three times a week. Her grades are mostly B's. She admits that she started to be sexually active for the past six months and has delayed periods for two months now. She had her menarche at 12 years old and has regular periods. She also smokes a quarter of a pack of cigarettes per day. Which of the following is an adverse pregnancy outcome due to maternal smoking? Hyperbilirubinemia Large for gestational age Placental abruption Respiratory distress syndrome

Correct Answer ( C ) Explanation: Cigarette smoking during pregnancy is the most important modifiable risk factor associated with adverse pregnancy outcomes. Smoking and secondhand smoke exposure increase the risk of infertility, placental abruption, preterm premature rupture of membranes (PPROM), and placenta previa. Cigarette smoking also increases the risk of placental abruption. Dose-response curve analysis has consistently revealed that the risk of abruption is greatest among heavy smokers. Because PPROM is associated with both cigarette smoking and placental abruption, the relationship between cigarette smoking and abruption may be partially explained by the increased risk of PPROM. However, cigarette smoking appears to be a risk factor for placental abruption, independent of PPROM. Cigarette smoking has also been consistently associated with placenta previa. It is recommended that all pregnant women should be asked regularly about tobacco use. In clinical practice, screening for tobacco use is done by asking the patient if she has ever smoked cigarettes, if she smoked when she found out that she was pregnant, and whether she smokes now. Women who smoke should be asked the number of cigarettes smoked per day. Hyperbilirubinemia (A) and respiratory distress syndrome (D) are not related to maternal smoking. Large for gestational age (B) is not an adverse pregnancy outcome from maternal smoking. Smoking in pregnancy results in the reduction in birth weight. The birth weight deficit associated with smoking is 100 to 300 grams, depending on the number of cigarettes smoked.

A family brings in their 3-month-old infant who was just adopted from Honduras. They are not aware of the infant's birth history and were told that the child was healthy and lived in an orphanage for the first three months of life. They are concerned that he has significant bruising over his back. On examination, a large blue patch is present over the sacrum. Similar patches are present over the bilateral shoulders. The remainder of the examination is benign. Which of the following is the most likely diagnosis? Child abuse Coagulopathy Dermal melanosis Nevus of Ota

Correct Answer ( C ) Explanation: Dermal melanosis, also known as a Mongolian spot, represents a benign persistence of dermal melanocytes in neonates. It presents as large blue patches that are most commonly located over the sacrum and shoulders. Rarely, the face and flexor surfaces of the extremities may be involved. It is more common amongst all non-Caucasian races. The natural history of dermal melanosis is a gradual fade during the first two years of life. Most lesions resolve completely during grade school. Extra-sacral locations are more likely to persist into adulthood. Persistent lesions remain benign, and no treatment is indicated. Child abuse (A) should be suspected in any infant with significant bruising in unusual locations, as well as in any child who does not walk ("those who don't cruise, don't bruise.") However, the distribution and description of this child's blue patches are consistent with dermal melanosis rather than child abuse. Coagulopathy (B) is a consideration in any child with unusual ecchymoses, purpura, petechiae, or bleeding. However, this infant's bluish discoloration over the sacrum and shoulders are consistent with dermal melanosis rather than true bruising. Nevus of Ota (D) is a rare dermal melanocytosis that preferentially involves the distribution of the first two trigeminal nerves. In contrast to dermal melanosis, its appearance is speckled or mottled and may be grey or blue-black in color. Underlying CNS melanosis or ocular melanoma may also occur, necessitating yearly ophthalmologic examination.

You examine a five-year-old girl for a routine child check. She likes to sing and dance. She knows her name and address. She can also draw a person with six body parts. She also eats fruits, vegetables, fish, and meat. The physical examination is normal. Her mother has no concerns. The girl is excited for the coming break because the family is going to the beach. Which of the following is the correct statement regarding sun exposure? Sunburn is primarily due to UV-A exposure Sunscreen should be applied generously five minutes before going outdoors Sunscreen should not be used on infants < 6 months of age Use a broad-spectrum, water-resistant sunscreen product with a minimum SPF of 15

Correct Answer ( C ) Explanation: Discussion of sun exposure should be part of the routine well-child visit. Ultraviolet radiation (UVR) emitted from the sun is divided into three wavelength ranges namely, UV-A (320 to 400 nm), UV-B (290 to 320 nm), and UV-C (200 to 290 nm). Exposure to ultraviolet light, whether from the sun or via indoor tanning, is the most significant environmental risk factor for all types of skin cancer. Sunscreens work by absorbing radiation in the range of 290 to 400 nm by either physical or chemical mechanisms. Sun protection factor (SPF) is primarily a measure of UV-B protection and is defined as the least amount of UV radiation required to produce minimal erythema on sunscreen-protected skin to that required to produce the same erythema on unprotected skin. To reduce ultraviolet exposure, the American Academy of Dermatology recommends using sunscreens as a third line of defense, after clothing and shade. Patients should be encouraged to use a broad-spectrum, water-resistant sunscreen product with a minimum SPF of 30. Sunscreen should be applied generously (1 ounce) 15 minutes before going outdoors (every day, all year round) and reapplied every two hours, especially after swimming or sweating. Sunscreen should not be used on infants < 6 months of age. For this age group, shade and clothing should be used instead. Sunburn is primarily due to UV-A exposure (A) is wrong because it is due to UV-B. Sunscreen should be applied generously five minutes before going outdoors (B) is not correct because sunscreen should be applied 15 minutes prior to sun exposure. Using a broad-spectrum, water-resistant sunscreen product with a minimum SPF of 15 (D) is false because minimum SPF should be 30.

Which of the following patients is at highest risk for developing cholelithiasis? 21-year-old woman of European descent 35-year-old thin man with gastric ulcers 41 year-old obese, pregnant woman 55-year-old man on a statin medication

Correct Answer ( C ) Explanation: Gallstones, or cholelithiasis, occur in >10% of American adults. They are made up of cholesterol, bile and phospholipids. The two main types are cholesterol stones (90%) and pigmented stone (10%). Cholesterol stones can exist as a single stone or multiple stones. Black pigmented stones can arise in patients with hepatitis and cirrhosis, while brown stones occur during bile duct infection and stasis. Risk factors include age over 40 years old, female sex, pregnancy and obesity. Biliary colic is the main symptom, and is described as an acute onset, episodic right upper quadrant or epigastric pain and nausea, which lasts for 30 minutes to 3 hours. Diagnosis is made via ultrasound. Treatment includes cholecystectomy and ursodeoxycholic acid. Gallstones are more common in South, Central and Native Americans, not Europeans (A). Gallstones occur more often in those with ileal disease, like Crohn's, not gastric disease (B). Statins (D) may decrease, not increase, the risk of gallstone formation.

A 66-year-old woman has been on estrogen-replacement therapy for 9 years. Her social history is significant for 20 years of alcohol abuse and 30 years of tobacco use. Her family history is positive for familial adenomatous polyposis. Which of the following organs is most likely to undergo malignant transformation due to this patient's alcohol abuse? Breast Colorectal Liver Lung

Correct Answer ( C ) Explanation: Hepatocellular carcinoma, also known as malignant hepatoma, is a primary cancer of the liver. It is rare in the United States, where most of liver cancer is secondary from a metastatic site. However, in countries where hepatitis is endemic, such as China, this cancer represents the most common of all liver cancers. The most common causes are viral hepatitis (hepatitis B and C) and alcoholic cirrhosis. Other risk factors include aflatoxin exposure, hemochromatosis, Wilson's disease and type 2 diabetes mellitus. Risk of developing this cancer in someone infected with hepatitis B or C is 20-25%, while those with alcoholic cirrhosis have upwards of an 80% chance of developing hepatocellular carcinoma. Alpha-fetoprotein is elevated in the majority of cases but it is nonspecific. Triple or quad-phase liver CT scan aides in the diagnosis. Biopsy provides a definitive diagnosis. Ultrasound plays an important role in the surveillance of high-risk patients, however, contrast enhanced abdominal CT with triple phase scanning remains the primary diagnostic tool. Treatment options include surgical resection, transplantation, chemotherapy and radiotherapy. Prolonged estrogen-replacement therapy is a major risk factor for breast cancer (A). Up to 25% of patients with colorectal cancer (B) have a family history of familial adenomatous polyposis. Tobacco use is a major risk factor for lung (D), not liver, cancer.

A 12-month-old boy presents with a hoarse, harsh cough and inspiratory stridor following two days of mild cough and nasal congestion. He is in mild respiratory distress. Lungs are clear to auscultation. What is the most likely diagnosis? Bacterial tracheitis Epiglottitis Laryngotracheitis Retropharyngeal abscess

Correct Answer ( C ) Explanation: Laryngotracheitis, also known as croup, is a common cause of inspiratory stridor in children. Croup is common amongst children ages 6 to 36 months and is most commonly caused by parainfluenza virus. The illness typically begins with one to two days of non-specific symptoms such as cough, nasal congestion, and low-grade fever. Children then may develop significant hoarseness, a barky cough, and inspiratory stridor. The stridor may occur only with activity or may occur at rest. A classic "Steeple sign" is present on x-ray, but x-rays are unnecessary for establishment of a diagnosis. The mainstay of treatment for croup is a glucocorticoid such as single-dose dexamethasone. Nebulized racemic epinephrine can be used for a rescue treatment for children with respiratory distress. The cough typically resolves within three days, but other symptoms may persist for a week. Bacterial tracheitis (A) is an exudative inflammation of the trachea and is a rare pediatric diagnosis. It is most common in children less than six years of age and often follows a viral upper respiratory infection. The onset of bacterial tracheitis more rapid and severe than croup, often necessitating airway support. Fever and toxic appearance are characteristic, but a barking cough is atypical. Similar to croup, patients with bacterial tracheitis may have a "steeple sign" on neck films, and thus radiographs cannot be used for a definitive diagnosis. A definitive diagnosis is established by endoscopy, which reveals inflammation and suppuration of the trachea. Epiglottitis (B) also has a more acute and severe onset than croup. Similar to bacterial tracheitis, epiglottitis often presents with severe respiratory distress, drooling, and fever. A barking cough is not typical. Patients with epiglottitis characteristically appear toxic and assume the tripod or sniffing position to maximize airway patency. Although epiglottitis is rare in the era of routine Haemophilus influenzae vaccination, it should be considered in any patient with the acute onset of severe upper respiratory tract obstruction. The classic radiographic finding of epiglottitis is the thumbprint sign. Deep neck space abscesses, such as a retropharyngeal abscess (D), often present with drooling and neck extension. Approximately one half of cases are associated with a preceding upper respiratory tract infection, but a barking cough is not characteristic. Stridor may develop as swelling progresses. In infants and children, the retropharyngeal space contains two chains of lymph nodes that drain the nasopharynx, adenoids, middle ear, and Eustachian tube. These lymph nodes may suppurate and eventually lead to a retropharyngeal phlegmon and then abscess. Lateral neck films show a widened prevertebral space.

A 13-year-old boy presents with right knee pain. The pain increased gradually two days ago and has been persistent since that time. He has played soccer for several years but denies injury or trauma. He has been recently healthy except for a diarrheal illness two weeks prior, but he has no further diarrhea and no longer has a fever. His examination reveals a small right knee effusion without erythema, warmth, or significant tenderness. He has a mild limp, but examination is otherwise benign. What is the most likely etiology of his knee pain? Leukemia Osgood-Schlatter Disease Reactive arthritis Septic arthritis

Correct Answer ( C ) Explanation: Reactive arthritis refers to arthritis that develops secondary to cross-reactive antibodies from a recent infection. The most commonly associated infections include enteric infections, such as Campylobacter jejuni or Shigella dysenteriae and genitourinary infections such as Chlamydia trachomatis. Reactive arthritis occurs one to four weeks following the inciting infection. The arthritis is an asymmetric oligoarthritis that typically affects the lower extremities. Enthesitis may also occur. Associated symptoms classically include conjunctivitis and urethritis ("can't see, can't pee, can't climb a tree"). Mucosal lesions and rashes may also be present. The initial treatment of reactive arthritis is non-steroidal anti-inflammatory medications. Those with inadequate pain relief may require intra-articular or systemic glucocorticoids. Leukemia (A) can present with bone and joint pain, particularly at night. Leukemia remains on the differential for this child, but given the proximity of his arthritis to his recent diarrheal illness, reactive arthritis is more likely. Osgood-Schlatter Disease (B) is an overuse syndrome that results in apophysitis of the tibial tuberosity. It is bilateral in 25-50% of patients. Osgood-Schlatter Disease presents with the gradual onset of anterior knee pain that is worsened with activity and improved with rest. Examination would reveal localized tenderness and swelling of the anterior tibial tuberosity with reproduction of pain upon squatting or extending the knee against resistance. Arthritis is not a finding of Osgood-Schlatter Disease. Septic arthritis (D) is a bacterial infection of a joint. In contrast to reactive arthritis, septic arthritis presents with acute findings of joint inflammation such as warmth, swelling, tenderness, and erythema. A fever is often present. Septic arthritis requires emergency drainage to prevent permanent joint damage. Rapid Review Reiter Syndrome (Reactive Arthritis) Patient with a history of recent GI or Chlamydia infection Complaining of acute asymmetric arthritis PE will show conjunctivitis, arthritis, urethritis Labs will show HLA-B27 Treatment is NSAIDs Comments: "Can't see" (uveitis), "can't pee" (urethritis), "can't climb a tree" (arthritis)

A one-year old boy presenting for a well-child visit is found to have asymmetric corneal light reflexes. His pupils are symmetric in size and shape, and a normal bilateral pupillary light reflex is present. His mother reports that sometimes his left eye appears to turn inward by itself when he is looking directly at her. You notice that intermittently the child's left eye appears to deviate medially in comparison to the right eye. Which of the following is the most likely diagnosis? Amblyopia Anisocoria Esotropia Exotropia

Correct Answer ( C ) Explanation: Strabismus is the misalignment of the visual axes of the eyes. Esotropia is a form of strabismus characterized by convergent axes, while exotropia is characterized by divergent axes. The child in the above clinical scenario has esotropia as suggested by the eye turning inward, as well as the asymmetry of the corneal light reflex. Intermittent esotropia may occur in normal infants up to 5-6 months of age. Esotropia is more serious than exotropia and often requires surgical correction. Exotropia may initially be treated with patching and exercises but may also need surgical correction. Complications of strabismus include amblyopia (decreased visual acuity in the less dominant eye), diplopia (double vision), and contracture of the extraocular muscles. Acquired strabismus (occurring after the first year of life) is of more concern and may be the result of a significant visual deficit or CNS disease. Amblyopia (A) is a condition of decreased visual acuity caused by visual deprivation or abnormal binocular interaction, and is classified based on the clinical condition responsible for its development. Untreated conditions of strabismus such as an esotropia, can lead to the development of amblyopia. Anisocoria (B) refers to a difference in size (of at least 0.4 mm or more) between the pupils; it may be the result of a benign or in some cases, very serious underlying condition. An exotropia (D) is a type of strabismus where the gaze deviates outwardly.

A 13-year-old boy is in the emergency department complaining of left scrotal pain and nausea that started two hours ago. Physical exam is significant for exquisite tenderness of the left testicle and no cremasteric reflex can be elicited. Ultrasound with Doppler shows decreased blood flow to the left testicle. What is the most likely diagnosis? Epididymitis Inguinal hernia Testicular torsion Varicocele

Correct Answer ( C ) Explanation: Testicular torsion is a surgical emergency that results from the twisting of the spermatic cord, which leads to reduced blood flow to the testicle. Timing of symptoms is very important because eight to 12 hours after onset increases the likelihood of significant ischemic damage. The physical exam should focus on the abdomen and inguinal area to rule out other causes of scrotal pain, such as an inguinal hernia. Cremasteric reflex is usually absent in patients with testicular torsion, but it does not rule out torsion, if present. Ultrasound with Doppler is usually done to evaluate blood flow if testicular torsion is suspected. A urinalysis should also be done to look for pyuria and bacteriuria, which is more consistent with epididymitis. Manual detorsion can be attempted if the patient's pain is controlled and tolerable. This is attempted via the "open book" rotation technique. If successful, there will be sudden pain relief, the testicle will change orientation, and it will drop lower in the scrotum. Epididymitis (A) presents with scrotal pain and swelling that is less severe and usually associated with an infection. Inguinal hernia (B) that is incarcerated would present with severe pain and an irreducible mass in the groin. Varicocele (D) is caused by a dilatation of the scrotal venous system. They can be asymptomatic or present with a dull ache. A soft mass can sometimes be palpated and has been known to feel like a "bag of worms."

A 46-year-old woman is in your office for her yearly physical. What is your interpretation of her ECG? Atrial fibrillation Atrial flutter Normal sinus rhythm Sinus tachycardia

Correct Answer ( C ) Explanation: The ECG is representative of normal sinus rhythm with a rate of 80. Normal sinus rhythm has a rate between 60-100. The rhythm is regular with a 1:1 relationship of the P to QRS. The PR interval is 120-200 msec, the QRS complex is 60-100 msec. P waves are upright in leads I, II, and aVF. Atrial fibrillation (A) is an irregularly irregular rhythm due to uncoordinated atrial activation and random occurrence of ventricular depolarization. The atria are not contracting but they do discharge electrical impulses to the ventricles. However, no single impulse depolarizes the atria completely, so only an occasional impulse gets through the AV node. It is the most common sustained dysrhythmia in clinical practice. Atrial flutter (B) is a rapid atrial rhythm but due to nodal delay, ventricular response rate is slower. Therefore, atrial flutter always occurs with some sort of AV block so that not all impulses are conducted. The resulting block is often variable (2:1, 3:1, 4:1). P waves have characteristic sawtooth pattern. In sinus tachycardia (D) the SA node is the pacemaker that causes the atria to depolarize regularly and, thus, the ventricles to depolarize regularly. P waves and QRS complexes occur regularly and the rate is >100.

An otherwise healthy 16-year-old boy is seen in clinic for a routine physical examination. He is a straight-A student and does not participate in sports. On exam you note that his height is greater than the ninety-fifth-percentile for age, he has long fingers, a pectus deformity, and auscultate a mid-systolic click followed by a 3/6 late systolic murmur. Which of the following is the most likely diagnosis? Ehlers Danlos syndrome Homocystinuria Marfan syndrome Russell Silver syndrome

Correct Answer ( C ) Explanation: The boy has a constellation of findings that are most suspicious for Marfan syndrome. Marfan syndrome is an autosomal dominant disorder caused by the abnormal biosynthesis of fibrillin-1. It is a multisystem disorder with cardinal manifestations in the skeletal, cardiovascular and ocular systems. The skeletal deformities include disproportionate overgrowth of long bones, anterior chest deformity (pectus carinatum or pectus excavatum), reduced upper to lower segment ratio, joint hypermobility, and arachnodactyly. Common cardiovascular manifestations are prolapse of the mitral or tricuspid valves. Individuals are at increased risk for an aortic aneurysm or dissection or both. Aortic abnormalities are the most life-threatening which require lifelong monitoring by echocardiography. Ectopia lentis or dislocation of the ocular lens is the most common ocular abnormality that is present in 60-70% of patients. Diagnosis of Marfan syndrome is based on clinical criteria and molecular analyses. Management involves a multidisciplinary team due to the complexity of the condition. Ehlers Danlos syndrome (A) has many similarities with Marfan syndrome which includes joint hypermobility, mitral valve prolapse and aortic dissection. The findings that differentiate it are skin laxity, contractures, skin nodules, and easy bruising. Homocystinuria (B) is a type of aminoacidopathy that can present with Marfanoid features, but patients also have neurologic and cognitive problems. Russell Silver syndrome (D) presents with dwarfism and hemihypertrophy.

What is the classic auscultatory feature of mitral valve prolapse? Crescendo-decrescendo systolic murmur Loud S1 and an opening snap in diastole Midsytolic click Soft diastolic murmur

Correct Answer ( C ) Explanation: The classic auscultatory features of mitral valve prolapse are a midsystolic "click" followed by a midsystolic to late systolic murmur over the mitral area. This click results from snapping of the chordae tendineae during the prolapse of the valve. Mitral valve prolapse (MVP) is defined pathophysiologically as an abnormal movement of one or both of the mitral valve leaflets across the plane of the valve during systole. The typical auscultatory findings should suggest MVP and can be confirmed by echocardiography. Symptoms attributed to MVP, however, are often not explained by the degree of prolapse or mitral regurgitation. Although generally a benign condition, it is infrequently associated with more serious cardiac pathology such as mitral regurgitation, endocarditis, and arrhythmias. Echocardiographic studies report a true prevalence of less than 1% in both men and women versus the previously reported 5% with a female predominance. Crescendo-decrescendo systolic murmur (A) is the classic auscultatory finding in aortic stenosis. In mitral stenosis auscultatory findings include a loud S1 and an opening snap (B) in early diastole accompanied by a low-pitched, rumbling diastolic apical murmur. A short, soft, diastolic murmur (D) may be difficult to detect an aortic insufficiency therefore an echocardiography may be required and is diagnostic.

A two-month-old infant presents with two weeks of nasal congestion and cough. Over the past two days, the cough has acutely worsened. Her family describes discrete episodes of coughing and gagging, sometimes followed by emesis. She appears well between coughing episodes. What treatment is indicated? Honey Intramuscular ceftriaxone Oral azithromycin Supportive care

Correct Answer ( C ) Explanation: The infant has a presentation consistent with Bordetella pertussis infection. Pertussis is spread through aerosolized droplets. After a seven- to 14-day incubation period, symptoms begin with nasal congestion, cough, and low-grade fever. This period, known as the catarrhal phase, lasts one to two weeks. Subsequently, the infant progresses to the paroxysmal phase, which includes episodic, "rapid fire," repetitive coughing. The coughing may include cyanosis, salivation, and lacrimation. The cough is classically followed by an inspiratory "whoop." The whoop is most common in infants older than six months of age. The whoop is uncommon among older children and adolescents. The intensity of coughing may also result in post-tussive emesis. The paroxysmal stage typically lasts for four weeks. The child will then progress to the convalescent phase, during which he or she will gradually have decreasing frequency and severity of coughing episodes. The convalescent stage may last for a few weeks to several months. Children with pertussis remain contagious until B. pertussis is eradicated from their nasopharynx. Spontaneous clearance typically occurs in two to four weeks. Oral azithromycin is the drug of choice for rapidly eradicating B. pertussis from the nasopharynx. Its dosing is 10 mg/kg for five days, which is notably different from recommended dosing for community-acquired pneumonia. Treatment will not result in symptom resolution, but the child will no longer be contagious. Intramuscular ceftriaxone (B), a third-generation cephalosporin, is not used to treat pertussis. It may be used to treat community-acquired pneumonia caused by typical pathogens such as S. pneumoniae, H. influenzae, and M. catarrhalis. Honey (A) is never recommended for infants due to the risk of infantile botulism transmitted by C. botulism spores. Honey can be used to soothe coughs in children older than 12 months of age. Supportive care (D) alone for pertussis is not recommended, as the child will remain contagious until B. pertussis is cleared from the nasopharynx. Azithromycin will hasten eradication of the organism and thus decrease the period of contagiousness.

A 16-year-old boy presents to the ED with a foreign body sensation and buzzing in his right ear that began early this morning and woke him up. Physical exam reveals a moving insect in the external auditory canal. Which of the following is the most appropriate next step in management of this condition? Discharge with ENT follow-up Foreign body removal with alligator forceps Instillation of lidocaine solution in the ear canal Irrigation of the ear canal with saline

Correct Answer ( C ) Explanation: The patient presents with a live insect foreign body in his ear canal. Insects are one of the most common otic foreign bodies in both adults and children. Patients with live insect foreign bodies in the ear canal often require immediate attention because of the extent of their discomfort and agitation. The first step in management is instillation of lidocaine solution in the ear canal to drown the insect. This step is crucial as it will increase patient comfort, prevent further trauma to ear structures caused by the insect, and make removal of the insect easier. It may also have an irritant effect on the insect, causing it to exit the ear on its own, as well as provide anesthesia for subsequent foreign body removal. Other liquids can be used to drown the insect, but these may not have an irritant effect and will not provide anesthesia. Once the insect is dead, attempts to remove it can follow. Many techniques and tools for otic foreign body removal exist, including indirect foreign body removal via irrigation of the ear canal with saline (D) or water as well as direct foreign body removal with alligator forceps (B), suction, right-angle hooks, balloon-tipped catheters, or glue-tipped swabs. Irrigation is contraindicated if the tympanic membrane is perforated or if the foreign body is vegetable matter, as this can swell in the presence of moisture and become more difficult to remove. Once the foreign body is removed, it is important to reinspect the ear canal for remaining foreign bodies and evidence of trauma or infection. Although discharge with ENT follow-up (A) may be necessary if complications arise or foreign body removal is unsuccessful, it is not an appropriate first step in management of this condition since the emergency physician can successfully remove most otic foreign bodies.

An otherwise healthy 4-year-old boy presents with his parents for vomiting and diarrhea. Several kids at his school have had similar symptoms. He began to have nonbloody, nonbilious vomiting yesterday and this morning began to have nonbloody diarrhea. He felt warm at home, but he is currently afebrile. He is well-appearing and playful. He is able to drink water without difficulty. Which of the following is the next best step? Administer intravenous fluids Admit for observation Provide reassurance and discharge home Send stool cultures

Correct Answer ( C ) Explanation: Viral gastroenteritis is a common illness in children. Globally, it is one of leading causes of pediatric deaths due to dehydration. Transmission is via the fecal-oral route. Once infected, patients will often have non-bloody, non-bilious vomiting and then develop watery diarrhea. The patient may have fever, and may have some abdominal pain. The duration of illness is typically less than 7 days. Dehydration is the biggest concern. Children who can successfully take oral liquids and appear well are safe for discharge home. The parents should be provided with reassurance, given instructions for oral hydration and red flag symptoms. Some children require pharmacologic control of their nausea. Oral ondansetron is the first line anti-emetic for gastroenteritis. A well-appearing child who is able to drink does not require administration of intravenous (IV) fluids (A). Oral rehydration is the recommended first line therapy. IV fluids often overused in industrialized nations. Routine stool cultures (D) are not indicated in acute gastroenteritis. Stool cultures should be obtained when there is persistent diarrhea, antibiotics are being considered, or when infection must be excluded. A well-appearing child does not need admission for observation (B) for uncomplicated gastroenteritis. Observation can be considered for children unable to drink fluids, hypoglycemia, or with moderate to severe dehydration.

Which finding on synovial fluid analysis is most consistent with the diagnosis of pseudogout? Elevated synovial leukocyte count Multiple gram-negative cocci Negatively birefringent urate crystals Positively birefringent calcium pyrophosphate dihydrate crystals

Correct Answer ( D ) Explanation: Calcium pyrophosphate crystal deposition disease, also referred to as pseudogout, is a common crystal-induced arthropathy that generally affects the large joints. Pseudogout has a similar clinical presentation to gout, but the etiology is different. Pseudogout may be idiopathic, especially in the elderly. It may also be caused by trauma, hyperparathyroidism, hemochromatosis, and medications that cause hypomagnesemia such as loop diuretics or proton pump inhibitors used in peptic ulcer disease. Patients present with acute onset of severe pain, inflammation and edema in the knees, ankles, elbows or wrists. Pseudogout is generally monoarticular, but may present in multiple joints as well. Diagnosis is by synovial fluid analysis, so arthrocentesis is necessary for patients with monoarticular arthritis. Synovial fluid analysis will show positively birefringent calcium pyrophosphate dihydrate crystals, which confirms the diagnosis. Initial treatment for pseudogout is with nonsteroidal anti-inflammatory drugs (NSAIDs) or colchicine. Elevated synovial leukocyte count (A) is seen in septic arthritis. Multiple gram-negative cocci (B) aspirated from an inflamed joint points to the diagnosis of gonococcal arthritis, caused by infection with the sexually transmitted Neisseria gonorrhoeae bacteria. Negatively birefringent urate crystals (C) seen on synovial fluid analysis are consistent with the diagnosis of gout.

A 67-year-old man with diabetes mellitus, chronic lower back pain and previously stable systolic heart failure now has increasing orthopnea and shortness of breath. Physical exam reveals pulmonary crackles, jugular venous distention and lower extremity edema. He is on carvedilol, lisinopril, furosemide, insulin and ibuprofen. Which of the following is appropriate for management of this patient? Add a calcium channel blocker Decrease the dosage of furosemide Discontinue the carvedilol Discontinue the ibuprofen

Correct Answer ( D ) Explanation: Non-steroidal anti-inflammatory drugs (e.g. ibuprofen) can worsen symptoms of heart failure. In fact, a number of medications that are in common clinical use are relatively or absolutely contraindicated in patients with heart failure. This is either because they can cause exacerbations of heart failure or because there is a higher risk of adverse events in such patients. Mechanisms by which some of these drugs can exacerbate heart failure relate to sodium retention, negative inotropic effects or direct cardiotoxicity. Non-steroidal anti-inflammatory drugs are associated with an increased risk of heart failure exacerbation, increased renal dysfunction and impairment of the response of angiotensin converting enzyme inhibitors and diuretics. Observational data has reported an association with non-steroidal anti-inflammatory drugs and increased mortality in patients with heart failure. It would not be correct to add a calcium channel blocker (A) as this class of medication has no direct role in the management of heart failure. Some studies have shown that their negative inotropic activity cause a greater clinical deterioration when compared to placebo, particularly with the first generation, non-dihydropyridines calcium channel blockers. It would also not be appropriate to decrease the dose of furosemide (B) as this patient is volume overloaded and needs diuresis to help alleviate his symptoms. It would not be appropriate to discontinue carvedilol (C) at this time. Although, generally, starting beta blocker therapy is not recommended in patients with decompensated heart failure, it is okay to continue beta blocker therapy if the patient has been previously stable on this medication.

A 39-year-old woman in her third trimester presents with two days of bloody "spotting" on her underwear. This is her third pregnancy, which thus far has been uncomplicated. Her initial delivery was vaginal and her second delivery was via cesarean section. She is currently sexually active, and has a history of trichomoniasis. She denies pelvic pain. Laboratory examination reveals hematocrit of 32%, white blood cell count of 10,000, platelet count of 260,000 INR of 1.1 and aPTT of 32 seconds. Pelvic examination shows a nonerythematous cervix with clear mucus. Which of the following is the most likely diagnosis? Cervicitis HELLP syndrome Maternal coagulopathy Placenta previa

Correct Answer ( D ) Explanation: One of the common causes of third trimester bleeding is placenta previa, in which the placenta improperly implants over or in close proximity to the cervical os due to abnormal vascularization. Risk factors include increased maternal age, multiparity, prior placenta previa and prior cesarean delivery. Incidence in nulliparas is about 1 in 1500, and increases to 1 in 20 in grandmultiparas. Bleeding, which is variable in amount and maternal in origin, typically presents in the 29th-30th gestational week. There typically is no pain associated with the bleeding. Diagnosis is made via ultrasonographic examination. Treatment consists of hospitalization, bed rest, hemodynamic stabilization if necessary and observation with expectant management until fetal maturity occurs (typically >36 weeks gestation). Cervicitis (A) is commonly caused by sexually transmitted disease, however we know nothing of her current partner's status, her current sexual practices or if the trichomoniasis was ever properly treated. Cervicitis is also a cause of third trimester bleeding, however it is unlikely in this case due to the absence of vaginal discharge, absence of pelvic pain and normal cervical appearance. The CBC is borderline normal. HELLP (B) is an acronym for describing certain prepartum hypertensive women with hemolytic anemia, elevated liver enzymes and low platelet count, most of which is not present in this patient. Although the hematocrit is low, women can normally have a physiologic decrease in hematocrit during pregnancy (the so-called physiologic "anemia" of pregnancy) due to the expansion of plasma volume being greater than the increase of red cell mass. Maternal coagulopathy (C) is unlikely in a patient with a normal coagulation panel as above.

Which of the following medications is an anticoagulant? Aspirin Clopidogrel Prasugrel Rivaroxaban

Correct Answer ( D ) Explanation: Rivaroxaban is an anticoagulant. Rivaroxaban is one of the new oral anticoagulants, which are commonly referred to as a NOAC. Other examples include dabigatran and apixaban. These new oral anticoagulants have become available as an alternative to warfarin anticoagulation in non-valvular atrial fibrillation. Options for anticoagulation have been expanding steadily over the past few decades, providing a greater number of agents for prevention and management of thromboembolic disease including atrial fibrillation and deep vein thrombosis. The new oral anticoagulants directly target the enzymatic activity of thrombin and factor Xa. These provide additional options to traditional therapy of unfractionated and low molecular weight heparin and vitamin K antagonists such as warfarin. They are attractive as they are taken orally and do not require routine monitoring. Aspirin (A), clopidogrel (B) and prasugrel (C) are all antiplatelet agents, not anticoagulants. Both anticoagulants and antiplatelets are commonly referred to as the lay term "blood thinners" and are easily confused. Hemostasis and clot formation is extremely complex and involves several processes that involve platelet aggregation and the clotting cascade. Aspirin and dual antiplatelet therapy with aspirin and a platelet P2Y12 receptor blocker are very important for primary and secondary prevention of acute coronary syndrome. Clopidogrel, is the oldest medication and most commonly used. It requires hepatic conversion and has issues with genetically predisposed non-responders. Some patients have genetic mutations or lack the enzymes that break down clopidogrel to its active form. Newer agents, such as prasugrel have been recently approved and do not require hepatic conversion or have issues with non-response.

A 3-year-old boy is brought in by his mother for fever and right ear tugging for 2 days. He has also had 1 week of rhinorrhea on review of systems. His vaccines are up to date. His physical exam is remarkable only for a temperature of 38.8°C and a bulging and erythematous right tympanic membrane. What is the most likely pathogen? Haemophilus influenzae, nontypeable Moraxella catarrhalis Staphylococcus aureus Streptococcus pneumoniae

Correct Answer ( D ) Explanation: Streptococcus pneumoniae accounts for 35% of the cases of otitis media in children. Rapid Review Acute Otitis Media Patient will be an infant or young child Complaining of ear pain, fever, URI symptoms PE will show TM erythema and decreased mobility of TM Most commonly caused by viral > bacterial (S. pneumoniae most common) Treatment is amoxicillin The next most frequently isolated pathogens are nontypeable Haemophilus influenzae (A) with 25% of cases and Moraxella catarrhalis (B) with 15%. Although the incidence of Staphylococcus aureus (C) otitis media has increased after the pneumococcus vaccine, it is still a rare cause of otitis media.

A 65-year-old woman presented to the emergency department with sub-sternal chest pain and dyspnea. Her cardiac biomarkers were mildly elevated and there was ST-segment elevation in the anterior leads. Apical ballooning was seen on echocardiography and coronary angiography revealed normal coronary arteries. She has no past medical history and takes no medications. Recently the patient lost her husband of 40 years. What is the most likely diagnosis? Non-ST-elevation myocardial infarction Pericarditis ST-elevation myocardial infarction Stress-induced cardiomyopathy

Correct Answer ( D ) Explanation: Stress-induced cardiomyopathy, also called Takotsubo cardiomyopathy and "broken heart" syndrome, is an increasingly reported syndrome characterized by transient cardiac dysfunction with ventricular apical ballooning, usually triggered by intense emotional or physical stress. This syndrome mimics acute myocardial infarction, but in the absence of obstructive coronary artery disease. It is approximated that stress cardiomyopathy accounts for approximately 2% of suspected acute coronary syndromes. Stress cardiomyopathy was historically called Takotsubo cardiomyopathy. Takotsubo is a Japanese word for octopus trap which resembles the characteristic apical ballooning seen on ventriculogram or echocardiography. It has also been referred to as apical ballooning syndrome or broken heart syndrome. The pathophysiology remains unknown, but catecholamine-mediated myocardial stunning is thought to be the most likely mechanism. This disorder is more common in woman, specifically postmenopausal women. Patients typically present following physical stress or a stressful emotional event such as a loss of a loved one, natural disaster, or devastating financial losses; however, a triggering event is not always present. Common presenting features include electrocardiographic changes (often anterior ST-segment elevations), mildly elevated cardiac biomarkers, sub-sternal chest pain, and dyspnea. Accepted criteria for the diagnosis are (1) ST segment elevation, (2) transient regional wall motion abnormalities of apex and mid ventricle, (3) the absence of coronary artery disease, and (4) absence of other causes of left ventricular dysfunction such as pheochromocytoma or myocarditis. In-hospital mortality is approximately 2% and patients who survive the acute episode typically recover in several weeks. Treatment is largely supportive care with hydration and efforts to alleviate physical or emotional stressor. Distinguishing stress cardiomyopathy from a non-ST-elevation myocardial infarction (A) and ST-elevation myocardial infarction (C) is often difficult. Patients in both groups present with electrocardiographic changes and elevated cardiac biomarkers. The normal coronary arteries by coronary angiography suggest that this patient does not have either a non-ST-elevation myocardial infarction or ST-elevation myocardial infarction. Patients with pericarditis (B) usually present with pleuritic chest pain and different findings on electrocardiogram, including PR-segment depression and global ST-segment elevation. Additionally, wall motion abnormalities on imaging is not typical of acute pericarditis.

Which of the following is associated with an increased likelihood of testicular torsion? Age greater than 50 years Epididymitis Fixed testis Undescended testis

Correct Answer ( D ) Explanation: Testicular torsion can occur at any age, but it has bimodal peaks: the first year of life and at puberty (ages 12 to 18). Torsion results from maldevelopment of fixation between the enveloping tunica vaginalis and the posterior scrotal wall, as seen with an undescended testicle (cryptorchidism). Characteristically, the involved testis is aligned along a horizontal rather than a vertical axis. Frequently there is a history of an athletic event, strenuous physical activity, or trauma just before the onset of scrotal pain. The pain usually occurs suddenly, is severe, and is usually felt in the lower abdominal quadrant, the inguinal canal, or the testis. On physical exam, there is loss of the cremasteric reflex. This is observed in nearly 100% of patients with torsion. The cremasteric reflex is a superficial reflex observed in men. This reflex is elicited by lightly stroking the superior and medial part of the thigh. The normal response is an immediate contraction of the cremaster muscle that pulls up the testis on the side stroked. In the setting of testicular torsion, this reflex is absent. The testicle is usually tender and firm and the scrotum is swollen and tender. In cases with a high suspicion of torsion, the patient should be taken to the OR immediately by a urologist. If the diagnosis is equivocal, color Doppler and ultrasound is the best diagnostic modality. Torsion is bimodal (A) with most cases occurring during the first year of life or during puberty. Epididymitis (B) may mimic the symptoms of torsion but is more gradual in onset. It does not predispose to torsion. Nearly 50% of cases arise in men between the ages 20 and 29 years. If untreated, it can lead to orchitis, testicular abscess, and sepsis. Although fixed testes (C) can still torse, it is not associated with an increased likelihood of torsion.

A 12-year-old girl presents with three days of sore throat, fever, malaise, and headache. Which of the following clinical features favors a diagnosis of infectious mononucleosis over Strep throat? Absence of coryza or cough Anterior cervical adenopathy Development of a diffuse, rough, maculopapular, erythematous exanthem Development of a maculopapular exanthem after starting empirical ampicillin

Correct Answer ( D ) Explanation: The development of a maculopapular rash following administration of penicillin is a classic feature of infectious mononucleosis secondary to the Epstein Barr Virus (EBV). This "ampicillin rash" classically occurs in patients with EBV mononucleosis who receive treatment with a beta-lactam antibiotic (e.g. amoxicillin, ampicillin). The rash results from a Type III hypersensitivity reaction and resolves spontaneously after discontinuation of the antibiotic. Hepatosplenomegaly is only found in infectious mononucleosis and not Strep pharyngitis. Strep throat and mononucleosis share many clinical characteristics, but certain features may help distinguish between the two infections. Absence of coryza and cough (A) are classically associated with Strep throat, although infectious mononucleosis often also lacks coryza and cough. Tender anterior cervical adenopathy (B) is classically associated with Strep throat, while posterior cervical adenopathy is characteristic of mononucleosis. However, EBV may also cause generalized adenopathy. Development of a diffuse, rough, maculopapular, erythematous exanthem (C) is classically associated with Scarlet Fever secondary to Streptococcus pyogenes infection. However, patients with EBV infection may have a variety of exanthems, including a scarlatiniform eruption. Rapid Review Infectious Mononucleosis Patient will be complaining of low-grade fever, headache, malaise, severe fatigue PE will show mildly tender lymphadenopathy involving the posterior cervical chain, hepatosplenomegaly Diagnosis is made by heterophile antibody test (monospot test), generalized maculopapular rash following administration of amoxicillin Most commonly caused by Epstein-Barr virus Treatment is self-limiting, refrain from contact sports for four weeks post-infection

A 26-year-old woman with a known history of AIDS presents to the ED for strange behavior, according to her boyfriend. Reportedly, she complained of a headache for a few days prior and then began acting bizarrely. In the ED, she has a temperature of 38.5°C. Neurological examination is remarkable for word-finding difficulties accompanied by episodes of clanging and echolalia, along with decreased attention span, recall, and consolidation. A contrast CT scan of the brain reveals multiple ring-enhancing lesions without evidence of midline shift. Which of the following is the most appropriate next step in management? Consult neurosurgery for a brain biopsy Obtain an MRI Treat with dexamethasone Treat with pyrimethamine and sulfadiazine Treat with trimethoprim-sulfamethoxazole

Correct Answer ( D ) Explanation: This patient with AIDS and altered mental status most likely has cerebral toxoplasmosis, the most common cause of focal encephalitis in patients with AIDS. It is often accompanied by fever, headache, altered mentation, focal neurologic deficits, and seizures. It is caused by the protozoa Toxoplasma gondii. The initial diagnosis is based on history, physical, and head CT scan. The appearance of multiple ring-enhancing lesions on contrast-enhanced head CT scan is pathognomonic. Treatment should be initiated with pyrimethamine and sulfadiazine. Some regimens also include folinic acid. Although an MRI (B) and brain biopsy (A) may be needed during subsequent management, in the immediate treatment period it is most important to begin antimicrobial therapy. Treatment with dexamethasone (C) is reasonable in patients with cerebral edema from mass effect that can be seen on a CT scan. Given that there is no evidence of midline shift on the CT scan, corticosteroids are not indicated. Trimethoprim-sulfamethoxazole (E) is generally used for prophylaxis, and while it can be used for treatment, it is not considered first-line.

A 28-year-old woman with a history of pernicious anemia presents with significant depigmentation of the skin on her hands and forearms. She has no history of damaging events to the skin or prior dermatologic disorders. Which of the following interventions would be the best initial step in managing this patient? Local liquid nitrogen application Narrowband UVB therapy Oral PUVA therapy Topical tacrolimus

Correct Answer ( D ) Explanation: Topical tacrolimus 0.1%, applied twice daily, is the first-line therapy for patients with vitiligo affecting less than 20% of the patient's body. Vitiligo is suspected in patients with any hypopigmentation in the color of their skin compared to the surrounding skin areas. The underlying cause of vitiligo is the destruction of melanocytes, or pigment cells. Often, there is an autoimmune component to this destruction and it may be seen in conjunction with diseases such as pernicious anemia, autoimmune thyroid disease, Addison disease, and type 1 diabetes. In the absence of an underlying autoimmune condition, other causes of hypopigmentation may be responsible. A history of using intralesional corticosteroids or liquid nitrogen over the affected skin may cause temporary loss of pigmentation, particularly in patients with a naturally olive or dark complexion. True vitiligo should also be distinguished from post-inflammatory hypopigmentation, as is seen in patients with a history of inflammatory skin conditions. A Wood's lamp can be used to accentuate vitiligo, as the hypopigmentation following an inflammatory condition will not enhance readily. Treating vitiligo is often a long process of trial-and-error. In patients with less than 20% involvement, topical tacrolimus can be recommended. Super-potent topical steroid creams may be used as well, but with the additional risk of skin atrophy in prolonged courses. Patients with 20-25% involvement may benefit more from narrowband UVB or oral PUVA therapy. However, patients should be counseled that there is a risk of severe phototoxic reactions with PUVA therapy. Generally, the fingertips and genitals are most difficult to treat, while the face and chest can be expected to respond well. Years of treatment are usually needed. Importantly, vitiligo increases a patient's risk of developing skin cancers, and more thorough skin screening methods should be considered. Local liquid nitrogen application (A) is generally not recommended to treat the hypopigmentation seen in vitiligo. In fact, patients with naturally darker complexions are at risk of developing permanent hypopigmentation after using liquid nitrogen to treat other dermatologic conditions. Narrowband UVB therapy (B) and oral PUVA therapy (C) are recommended treatments for vitiligo. However, these are not considered first-line treatments in patients with less than 20% skin involvement, as they carry an increased risk of phototoxic reactions.

What level does the spinal cord terminate in adults? L1-L2 L2-L3 L3-L4 L4-L5

Explanation: The level of the spinal cord varies by age as the spine grows. Knowing the level is essential knowledge prior to performing a lumbar puncture. In the fetus the spinal canal and spinal cord are equal in length. But, growth of the spinal cord does not keep pace with the growth of the spinal canal. In adults and older children the spinal cord (conus medullaris) terminates at the lower level of L1 spinal nerve or the body of L2. Therefore, the best approach for a lumbar puncture is between the L3-L4, L4-L5 or L5-S1 interspace. In infants, the cord terminates at L3 (B and C). Therefore, in infants a spinal needle should be placed in the L4-L5 (D) or L5-S1 interspace.

A 9-year-old boy presents with penile pain. On genitourinary exam the distal prepuce cannot be reduced over the glans penis and there are thin adhesions to the glans, which is swollen. What is the next best step in management? Dorsal penile slit procedure Emergency circumcision Manual reduction after topical lidocaine Topical corticosteroids

orrect Answer ( C ) Explanation: Attempt at manual reduction is indicated for paraphimosis which describes the inability to reduce the foreskin over the glans penis to its natural position. It is a medical emergency and can cause venous engorgement and glans edema which can lead to ischemia. It most often occurs when the foreskin is forcibly retracted to clean the penis and the foreskin is not returned to its natural position. Boys younger than age 4 are particularly susceptible to paraphimosis because their foreskin is not fully mobile. To manually reduce the paraphimosis, pain should first be controlled and then the foreskin should be circumferentially compressed to force the edematous fluid along the penile shaft. Then, the foreskin is pulled back over the glans. If manual reduction is not successful, an incision at the dorsal region of the constricting band (dorsal slit procedure) (A) should be performed to release the foreskin. Circumcision (B) should be performed at a later time. Topical steroids (D) are not indicated in paraphimosis but are helpful for mild penile adhesions.


Conjuntos de estudio relacionados

Series 7 New Issues Market Quiz Mistakes

View Set

Principles of Accounting and Finance Exam 4

View Set

โลกาภิวัตน์และสังคมวิทยา globalization & sociology

View Set

Chapter 49: Management of Patients with Urinary Disorders

View Set